You are on page 1of 43

ON the edge of society

Introductory Questions
Credits to my excellent teammate, Caitlin for completing some parts of this
resource!
 What does it mean to belong?

 In terms of the actual dictionary definition, “belong” is a verb that refers to the state of being
someone or something’s property. A secondary definition is when someone or something is “a
member or part of (a particular group, organisation or class). Nowadays the meaning of the word
is about as elastic as the WSC is when it comes to punctuality. Belonging is more commonly used
in society, where people are excluded or included because of traits that make them “belong” within
that social class.

 What does it mean for a group of people to be marginalized?

 Once again we refer to the dictionary first (as every logical thinker should probably do) in order
to answer this question. Marginalized is a term that refers to the treatment of people or groups in
ways that make them feel insignificant or peripheral. In this day and age, that term is used to refer
to the treatment of groups who are shunned from normal society and are often left on their own.

 Is it always better to be included than to be excluded?

 Would you rather be included in a group of bullies than be excluded? It depends on what group
you’re being included in and what group you’re being excluded from.

 Who decides who belongs in a certain group?

 The people inside the group itself often do. A common school example is the “popular people”,
who garner attention from everyone and are often defined by their physical appearance,
intelligence or other notable characteristic (including the apparent necessity to take alcohol and
other dangerous substances in the popular group of my school, which I am very much not in thank
goodness).

 Is it always wrong to exclude people from a group?


 Once more: is it wrong to exclude a kind-hearted athletic person from a group of athletic yet cold-
hearted people? It depends on the group and whether inclusion brings more harm than exclusion.
In the case of societies nowadays, we find that people consider it wrong to exclude minorities or
other oppressed groups from the general public.

 Are there any situations in which one might want to be marginalized?

 None that spring to mind, though perhaps a situation where people prefer not being noticed and
being excluded from society would bring this about. Indeed I doubt the uncontacted tribes living
deep in the Amazonian rainforest or remote Pacific islands would enjoy it if society suddenly
included them in our advancements.

 Should people ever be integrated with others against their will?

 One word: colonisation. The process literally involves integrating entire nations worth of people
into another group against their will, just to expand the power and territory of the group that
colonised (great job Europe, your empires are now to blame for marginalisation). In the modern
era people shouldn’t have their will forcefully taken away when it comes to integration, there are
those who prefer being recognised as separate from other groups and we should respect their
choice.

 What is the difference between exclusion and inequality?

 Exclusion refers to the act of making someone or something excluded, denying them access to a
group, place, privilege or other status that those included possess. Inequality refers to the state of
being unequal, when groups have vastly different privileges, statuses or other aspects that make
one more favorable than the other (we do not know of this term in glorious Motherland fellow
comrade!). When it comes to marginalisation, inequality refers to how much say, public attention,
privileges or other rights are unequally balanced amongst groups.

 Can people ever be “separate but equal”?

 Let us take a look at the exact origin of that term. In order to do that we must look at that vile land
that brands itself as the “land of the free” (America, for those of you not politically or historically
aware). “Separate but equal” referred to a legal doctrine in the United States Constitution which
stipulated that racial segregation did not violate the Fourteenth Amendment (equal protection
under the law to all people, regardless of race or class). In 1896 the court case Plessy vs. Ferguson
ruled that segregation was not discrimination so long as the separate facilities were equal (which
they were clearly not!). In the later half of the 20th century, this came under fire from social
activists calling for the removal of segregation, it was then that the court realised how unequal the
facilities were, causing for the doctrine to be scrapped.

 In the modern age, people can’t really be “separate but equal”, the two somehow appear to be
mutually exclusive. By separating groups and minorities, we are already making them unequal in
status to the larger public. By making them separate, we are giving them less attention and often
fewer rights than the general public along with many more inequalities. Separate but equal should
probably be replaced with “United and Equitable” for the modern era in my opinion.

 Are there any valid arguments against inclusiveness as a social goal?

 Not to my knowledge (which is limited in this field, I’m not exactly a major social activist). One
possible argument might be the risk of assimilation instead of inclusion. The more we integrate
groups and minorities into society, the more likely we are to strip them of what made them unique
in the first place and make them more like us (sort of an “us vs. them” standoff where one group
will siphon all the unique things off the other). Another argument might be that some groups would
prefer remaining separate from general society (this is more applicable towards actual cultural
groups than social niches nowadays).

 Are there any steps toward increasing inclusiveness with which you would be
uncomfortable?

 Forcefully including people in society is probably not something we should be aiming for, neither
is shunning other minorities in the interests of increasing the inclusiveness of others.

 Are there times when stratification is necessary or beneficial for a society?

 Firstly a quick definition: stratification refers to the process of forming layers, classes or categories
(more commonly used in geology to refer to topographical layers). You might recall a sort of
feudal system or socioeconomic class structure where people in a society were organised based on
their wealth, income or job (something Communism attempts to abolish, for the betterment of
society!). For much of history, stratification has been an inevitable aspect of society (even,
regrettably in the USSR). Capitalist consumer policies and government structures meant that those
who couldn’t afford a lot were at the bottom, working their backs off to feed the coffers of those
at the top. In the modern age we try and avoid social stratification, whereby groups are regarded
in lower lights and given lesser rights due to their characteristics. One of the biggest problems is
with females in society, some countries in the Middle East finds the concept of gender equality
outrageous whilst many western countries find the concept appealing for their future.

 Is the world becoming more inclusive? Is your school? Is your country?

 The world is slowly becoming more inclusive, my school as well and my country too (shortest
answer I’ll ever give in this subject area!).

 Is there a difference between being in the minority and being marginalized?

 There is indeed a difference (though nowadays the two are used interchangeably), being in the
minority refers to being part of something that makes up a lesser part of a whole than the other
parts (for example, minority cultures often have fewer people than the other cultures in the whole
society). While most minorities are marginalised in their treatment, the two are very different and
should not be confused.

 Are different marginalized groups in the same society natural allies, or are they just as likely
to turn on each other?

 Both. While these marginalised groups are often capable of coming tomorrow and uniting to ensure
society hears them, there is always the chance that they can turn on each other and cause the further
marginalisation of one group if they have conflicting thoughts/interests/beliefs.
People of a Feather: The Sources and Consequences of
Groups
 Basic Features of Social Groups

 power structures | roles | communication structures

 Power structures: A power structure is essentially some sort of societal construct that defines the
relationship between some individuals and other individuals within that power structure. This
mainly deals with how power and authority are distributed within a group, with a plutocratic power
structure having a small amount of wealthy people wield the power whilst absolutist structures
mean the government can wield all the authority and subjugate its subjects as brutally as it wishes.

 Roles: Roles in society encompass a wide variety of topics and areas. In order to better understand
the idea of roles, we need to delve into some of them more specifically. Economic roles often refer
to commonplace workers, CEOs and other employable positions within a business. Political roles
can refer to bureaucrats, lawyers, politicians and nation leaders. In society at the normal, citizen
level, roles often refer to who holds the power based on several factors. Marginalised people don’t
have large roles in society, since they aren’t recognised fully or hold much power in the
community. I wholeheartedly suggest that you further research these specific roles and what
obligations them come with on your own, as that takes far too much time for me to detail for every
single role.

 Communication Structures: If you thought about some sort of antenna that sends and receives
radio waves for our communication, you’re just ever so slightly off the mark (no not really, you’re
very wide off the mark). Simply put, a communication structure is the “pattern of interaction that
exists within a society”. Within societies and their divisions, there tend to be certain interactions
that are more common than others. For example, within the basic family structure, a mother is
more likely to interact with the child since the father is usually off to work for most of the day (this
may change as the child grows or the father retires, but this general trend exists practically
everywhere). Another example is how marginalised groups tend to get less communication to other
members of society because they are shunned from public interaction much more than other
groups.

 similarity | interdependence | injunctive and proscriptive norms

 Similarity: Simply put (apologies for the pun), similarity refers to how alike two (or more) things
are to one another. The more traits, characteristics, general interest and other aspects two things
share, the more likely they are to be closer to one another. Within society, this can be applied to
groups (or “niches”) of people. You’ve probably already seen this sort of phenomenon within your
school. New students tend to gravitate and befriend a group of people with whom they have a high
level of similarity. On the flipside, a lack of similarity can lead to public dissonance and societal
clashes, as is mostly the case with marginalised groups or minorities.

 Interdependence: “The dependence of two or more people (or things) on each other”.
Interdependence is sort of like the buddy cooperation of society. You rely on someone for
something and in exchange they rely on you for something else. A very common everyday example
is the workplace, where employees rely on their manager for income whilst their manager relies
on their employees to generate that income for the company. In school, this example is also
applicable within group work (assuming your group members actually do their work and don’t
leave everything to the smartest person, which is not what my amazing WSC team does!).
Interdepence is often ignored when it comes to marginalised groups on the edge of society, since
the main bulk of society doesn’t depend on them for anything (except perhaps, a scapegoat to
blame for some inexplicably linked sociopolitical issue).

 Injunctive and proscriptive: A legal term used in lawmaking and court cases (2 things I do not
enjoy discussing but shall do so for your enlightenment), injunctive refers to a judicial process
known as injunction which tells a certain party to refrain from doing an act. When a court orders
this remedy, it directs the conduct of that party, telling them what they can and cannot do as well
as setting out punishment or penalties if they infringe on the injunction. By contrast, a proscriptive
law refers to any sort of rule that forbids or restricts something. Outside of lawmaking, proscriptive
norms can refer to activities or opinions in society that a certain group should not do or have.
Injunctive norms can refer to people telling others what to do instead of giving them the free will
to do so.

 Entitativity: when does a group think of itself as a group?

 cultural identity groups | crowds vs. mobs

 Cultural identity groups: This should seem fairly simple, a cultural identity group refers to a
party of individuals who are together because their cultural background is similar to one another.
This includes racial groups (i.e black, white though those are less utilised nowadays), national
groups (i.e Asian-American, African-European) and geographical groups (i.e Pacific Islanders and
state-specific groups). These groups are very evident in society and a nation (or even cities) can
host a variety of cultural identity groups within it.

 Crowds vs. Mobs: Consider this analogy: A human who has yet to commit a crime is clean and
innocent in the eye of the law, yet a person who has committed crimes is now considered criminal
by the law. This sort of transformation applies to crowds and mobs. Crowds are simply a large
group of people within any space that are unorganised or chaotic. Often times, crowds are gathered
for specific reasons (i.e an audience at an outdoor concert, a group of people in a shopping mall or
even a political rally). A crowd is perfectly acceptable in society and it occurs everyday without
any protest. However, when a crowd turns violent or unruly and begins to cause harm to others,
then it becomes a mob. This might include violent political protests, a group of school bullies or
even a bunch of marginalised rioters. Mobs are where law enforcement officers are often sent in
and marginalisation has caused these occurrences to appear more often, with some even dying or
being critically injured thanks to their actions.

 Tuckman model | seceder model | Homans’ theory

 Tuckman model: Dr. Bruce Tuckman developed this sociological model in 1965 and later revised
it in the 1970s. Basically the Tuckman model outlines 5 stages that a group will go through the
longer that group stays together. The model states that as the maturity and ability of the team
grows, relationships are established and the leader changes their style to adapt to the new team
dynamics. Here are the 5 stages in slightly better detail:

 Forming: At this stage, the group has just been made and the members come together in order to
decide their goals and tackle tasks in order to achieve it. There is very little cohesion between
members and the leader still needs to give lots of directions and answer many questions about the
opportunities, goals and challenges ahead. Instead of acting as group members, they prefer to work
alone and make progress as individuals. Discussion within the group focuses on the specifics of
the task as well as how to align group members to both it and one another. To move onto the next
stage, all group members must be willing to let go of comfortable topics and tasks as well as admit
that conflicts will arise.
 Storming: By far the most comprehensive and most troublesome of the 5 stages, storming implies
a double meaning. Storming can refer to the massive amount of orientation and task completion
being done in this stage, as group members come to trust and confide in one another, resulting in
more cohesive and efficient group work. Storming can also however, refer to the “quest for power”
that the group might embark on, with some individuals attempting to secure closer positions to the
leader as possible. This can cause conflict to arise, especially when people call each other out on
their avoidance of responsibility or attempt to dominate group discussions. The group needs to
resolve these conflicts (perhaps by compromising) before it can move onto the next stage. Leaders
finding themselves in this stage need to redirect the team’s focus to completing the goal instead of
trying to fight one another, as well as coach specific individuals on their responsibilities and
relationships with other group members.

 Norming: Groups that reach the norming stage aren’t out of the storm just yet. Though the group
members have now developed relationships with one another, allowing for the asking of assistance
or receival of feedback, there can still be tensions as a result of new tasks that provide conflict. Its
during this stage that leaders need to be perceptive towards areas of conflict and resolve them
quickly, lest the team lapse back into the storming stage. It's here that the team gets to realise why
the leader was put in charge and respect for authority is introduced.

 Performing: A team at this stage is essentially fully mature and ready to face all its tasks with no
friction and hard work ethics. By this stage, the team is close to completing the entire objective
and fulfilling the goal, thanks to the processes and structure set up by the leader. Speaking of the
leader, during this time the leader should no longer need to direct or assist group members with
their tasks, instead they can safely delegate (assign) jobs to the group knowing that their input
beyond that should no longer be needed.

 Adjourning: Added in 1975 by Tuckman, the adjourning step isn’t exactly an extension of the
model, but rather an additional part to the fourth part once it has concluded. After the group’s task
has been fulfilled, they no longer need to remain together and the group terminates their
cooperation here. The leader is free to find another project to start a group on and the group
members can congratulate themselves as well as one another for reaching their goals and
objectives. However, at this time there can be friction, as insecure group members or those with
strong relationships refuse to part ways, perhaps forcing the group to remain together for the next
project.

 Seceder model: Alright then, here’s a model which has clearly been ignored in popular media,
since the most recent page I can find for it is the original publication from Cornell University in
2003. Simply put, the model shows how the desire to be different from the “average” in a
population can lead to the creation of more groups with in that society. As the “average” continues
to shift depending on the definition of the word by the sub-groups, even more groups are created
with the desire to be different from the rest.

 Homans’ theory: Homans’ Theory, otherwise known as a theory of group formation was created
by behavioral sociologist George C. Homans about the dynamics of how groups are created in the
first place (before Tuckman’s model can come into play). Homans’ theory of group formation
simply states that there are three key elements when it comes to forming a team: activities,
interactions and sentiments. It is these three elements that have knockback effects on one another,
as Homan stipulates in his work. The assigned activities are the tasks a person has to work on,
which can impact their interaction with other group members who might have sentiments for their
tasks and abilities.

 social exchange theory | swarm behavior | herd mentality

 Social exchange theory: Imagine if an economist suddenly decided to perform risk-benefit


analysis on every single human relationship they could find; that is essentially the concept of the
Social Exchange Theory. According to the theory, our interactions are fueled by our personal need
to find gratification from others and as such are determined by the rewards or punishments that we
receive. Oh and by the way, our old friend George Homans is credited with the discovery of this
theory. The central thought process behind the Social Exchange Theory is that an interaction which
produces more rewards will likely continue to occur, whilst an interaction that produces more
punishments will likely cease to continue. In fact, George Homans (along with later psychologists
such as Peter Blau or Richard Emerson), defined a formula of sorts to calculate the likelihood of a
repeated interaction in any situation. Here it is: behavior (profits) = rewards of interaction - costs
of interaction. If the end result is positive, then the interaction has been gratified and will be
repeated (vice versa if its negative). Before we move onto the more complex intricacies of this
concept, let’s have an example (and what better one than a romantic one!): Let’s say the social
interaction in question is asking someone out on a date. The cost is obviously that this interaction
may cause some communal outcry (especially if the person in question is leagues above you) but
the reward is obviously the fact that you’re going on a date (something I’ve never received the
reward of). If the person whom you asked out replies positively (a round of applause is in order),
then that gratification will likely cause you to repeat the interaction (either by asking them out
again or asking others out, though probably the former since the latter just makes you seem
disloyal). If however, (like me) the person responds negatively, then the cost will prevent you from
attempting this interaction anytime soon.

 Now then, let’s get into some of the more complex notions. The first is expectations: as you weigh
the benefits and costs of a future interaction, you will establish a comparison level based on
previous relationships or social norms. For example, if you’ve previously had poor relationships
with friends, you’ll likely expect that same level of support with interactions regarding your
friends. Another key aspect after the introduction of alternatives, where you assess the possible
paths other than the interaction and based on those analyses re-evaluate the cost-benefit of the
interaction.

 The final key concept within the Social Exchange Theory is the “Honeymoon phase” (no not that
period after a marriage you immature scholar). This refers to the length of a relationship and how
that influences your interactions as exchanges. During the first few weeks and even months of a
friendship, you’re more likely to ignore the social exchange evaluation and balance. Acts that seem
costly won’t be looked at, whilst potential benefits are often exaggerated in value. Once this phase
finishes, then a proper evaluation is usually performed, with the relationship terminating if the
negativity of it has been exposed (the costs are too high).

 Swarm behaviour: A collective behavior exhibited by animals and entities, this type of
phenomenon can easily be explained by the cliche (and sometimes false) quote “safety in
numbers”. Swarm behavior is shown whenever large flocks of animals migrate or (sorry) swarm
en masse to a certain location for some biological reason (i.e source of water, mating ground, food
source abundance). Humans often show this type of mentality by forming groups subconsciously
in crowded areas and moving in large formations to avoid being attacked or picked off. In a WSC
round, this might be your school delegation’s tendency to stick together throughout the transition
stages of the competition.

 Herd mentality: In 2008, professor Jens Krause and Dr. John Dyer of Leeds University performed
a sociological and psychological experiment. Groups of people were told to walk around randomly
in a great big room, with no communication between them. A select few were told the exact
directions and positions of where to walk. After some time, the researchers observed that those
walking “randomly” had begun to follow those who had been given directions (making it no so
random walking). This is herd mentality in practice. You are convinced by your peers to take a
course of action or adopt a behavior that may not be entirely rational or fitting for you. Instead this
is simply because your brain would rather follow others who are sure in their course of action than
risk the consequences of stumbling into unknown dangers. Common examples of this can be seen
everyday. Investors in stock markets often flock to buy shares from certain companies because a
large amount of their colleagues are doing so. Students may begin to follow certain trend in their
fashion choice or word choice because their peers are also doing so. Shoppers are more likely to
stick around in a store if they notice large crowds of other shoppers buying a certain object. In fact,
herd mentality has even been used to the benefit of individuals and businesses (it’s one reason why
this website has so many viewers, since many pass it on by word of mouth to others, who then use
it in mimicry of their teammates). Walt Disney actually used this to great effect when the first
Disneyland was opened. Visitors were simply leaving whenever their rides had been completed, s
to drive profits up Disney organised fireworks show (still ongoing on special occasions at all of its
parks!) and the herd simply followed the leader.

 To Follow the Group, To Go Astray

 informational vs normative vs referential conformity: As the last word suggests, all these terms
refer to ways in which we “conform” to groups or other parties in order to better “fit in” to avoid
friction or increase relationships. Conformity essentially involves caving in to peer pressure,
whether that pressure was applied forcefully or implied subtly. Informational conformity is the
most school-applicable one. This is where a person looks to a group or other individual because
they lack knowledge and seek the “correct” direction or information from others. In schools, this
might be exemplified by a student mimicking the behaviour of people they perceive to be more
intelligent, in order to get the right information from them (please do not mimic me scholar, it will
only bring you social embarrassment). Normative conformity is the more social type and often
deals with you changing your behaviour in order to fit in with the “norm” that society has set (this
can often mean changing your physical appearance or way in which you talk). Referential
conformity is slightly more obscure and refers to conforming to stereotypes or references of what
you should be based on your background.This is often concerned with race, nationality, religion
and other cultural aspects.

 social identity theory | self-categorization

 Social identity theory: Polish social psychologist Henri Tajfel is most well known for his
foundational work on this theory and it lays the cornerstone for many other branching theories
based on it. The Social Identity Theory stipulates that the groups which people belonged to in
society (or at least, felt they belonged to) generated a large amount of pride and self-esteem (it
could even shape their character). Our groups give us a sense of belonging in this world and it is
with them by our side that we can carry on in life. However, this social stigma does also come with
hidden dangers. In order to fuel the self-image, we tend to create “them” and “us” divisions
between groups, creating what is known as in-groups (us) and out-groups (them). If the tension
between the two groups reaches a boiling point, it can result in extreme racism and prejudices
against the out-group (as was the case in Germany between the Nazis and the Jews).

 Self-categorization: John Charles Turner can claim credit for this specific theory. Simply put, this
theory puts forth the processes by which people create their social identities by comparing their
own traits against those of other people both within and out of the social groups they’ve placed
themselves in. If you seek a more thorough understanding, I recommend visiting this website (be
warned though, the language can be a bit intimidating) for more specific terms that might appear
during the bowl or challenge.
 dominant culture | counterculture | high-brow vs. low-brow

 Dominant culture: As the name implies, a dominant culture is any sort of cultural practice that is
the dominant one in a society. It is this dominant culture that often controls the law, political stage,
educational institutes, business practices and even creative expression. A common example within
a nation might be that of the United States. It is within the dominant culture to speak English,
believe in a Protestant Christian religion and be of European ancestry. These cultures set the norm
for an entire society and are often indirectly responsible for marginalising cultures which do not
fit with them.

 Counterculture: Again, as the name implies a counterculture is a way of life that goes against the
norms set by society. At times this can be put on marginalised people or minority groups whose
cultural practices are not in line with what the prevailing public opinion (or dominant culture)
believes it should be. A common example of such a counterculture was the hippie subculture of
1960s and 70s America and Britain. Countercultures can even exist on something as small as a
school, with some individuals going against the norms of their peers (i.e me whenever it comes to
work ethics).

 Highbrow vs. lowbrow: These meaning are slightly harder to interpret, though a symbolic
analysis can easily divulge their intended message. Highbrow is a derogatory term used to refer to
“high culture”, essentially the valuing of works or practices with great intellectual significance.
Highbrows were people who followed such cultures and these often encompassed great thinkers,
snobby businessman and political leaders. In contrast, the common man might have more in
common with lowbrows, who were the people who didn’t mind “dirtier” cultures with less panache
and class than the highbrows. Modern society doesn’t really find a place for these words, since
culture nowadays can be an amalgamation of high and low culture (i.e music tastes can be low
culture whilst fashion taste might be high culture).

 Asch Paradigm | Crutchfield Situation

 Asch Paradigm: A recurring trend on this section is to include contributions from Polish
professionals and frankly their work is top-class when it comes to dissecting the human mind
(metaphorically, not literally). Solomon Eliot Asch was one such Polish person, yet he was also of
American origin. The Asch Paradigm for which he is most well-remembered is one of the greatest
lab experiments in social conformity. In 1951, Asch lined up 50 college students from Swarthmore
College in the United States to participate in his research. He lined up 8 of them along a wall, much
thee same way a police mug shot might occur (Brooklyn nine nine viewers, this scene may be
familiar to you). 7 of these people were actually “stooges” who had pre-discussed what response
they were all going to give (most of the time this response was incorrect). The remaining “naive
person” was to give their own response and the aim was to see whether or not they conformed to
the majority of group responses. What Asch found was astonishing: out of the 18 trials held, 75%
of participants conformed at least once, with an average conformation rate of 32% on the wrong
trials (called the “critical trials” for their value in the research). When interviewed post-experiment
why they had conformed, many participants gave the reply that they weren’t entirely sure of their
answer, but had gone along with the group in order to avoid being called “peculiar” or ridiculed
for their unique response. Some even believed that the group answer was the correct one and that
they had gone along on the basis of poor information (informational influence right here). The
main takeaway from the Asch Paradigm is that conformity is somehow natural in human beings
because we fear being singled out and picked at because of our uniqueness.

 Crutchfield Situation: What a most interesting study. Just 4 years after the Asch Paradigm was
conducted, American psychologist Richard S. Crutchfield felt the need to redo the experiment,
with some changes based on mass-criticisms the previous study had received. One of the largest
points of judgement was the unnatural condition that the confederates had to be large in number,
which Crutchfield neutralised by making sure there were no confederates in the actual experiment.
Another major point was how each participant had to announce their answer out loud, further
adding to the embarrassment on the naive person if they didn’t conform. So the participants in
Crutchfield’s situation were isolated from one another and instead of answering verbally, pressed
electronic switches. Now everyone else had their own lights that told them which switch the others
had pressed, but in reality this was controlled by the main experimenter from the circuit room
(adding an element of deception within the experiment). In fact, Crutchfield performed this
procedure with all the participants at the same time, thus the conformity behavior could be explored
in its simultaneous nature. As expected, while people still conformed, they didn’t do so in the
astounding numbers that Asch’s Paradigm did.

 Them-ocracy: Understanding Exclusion and Rivalry

 ingroup vs. outgroup | outgroup homogeneity | trait ascription

 Ingroup vs. outgroup: I refer to my answer on dominant culture

 Outgroup homogeneity: Ever consider people who aren’t in your social group to be essential
copies of one another? (I know I have). This is essentially your mind applying outgroup
homogeneity and its best explained using the phrase “they are alike, we are diverse”. It’s where
your brain believes that people of the out-group are more similar to each other than the people of
your in-group.

 Trait ascription: If you’ve ever considered yourself a mysterious person whose behavior cannot
be explained by some rational thought (i.e me at times whenever I get those Sherlock Holmes
influences), then this is likely your brain applying trait ascription to yourself. More formally
known as trait ascription bias, this is when you view yourself as variable in your behavior, mood
or personality and see others as predictable when faced with certain situations. You might think
you react differently for every event, but at the same time you’ll assign standard reactions for other
people based on their characters.

 intergroup interactions | social comparison | social invisibility

 Intergroup interactions: Literally what it says on the tin. Intergroup interactions refer to when
people from different social groups interact with one another, or when those groups collectively
come together and interact with each other. This can often create interesting, surprising and at
times frictional thoughts between the two. Perhaps if the popular kids in my school were to spend
an hour trapped with the nerds, some interesting things could happen. I’d much rather see the
antisocial people (myself included) locked up with the social creatures of my year for an hour
though, what torture that would be.

 Social comparison: Whenever you try and redeem your self-esteem or motivate yourself to
continue being in a group, you’ll often turn to evaluating your own position in life compared to
that of others. This is when your brain conducts social comparison(s), whereby it analyzes your
worth based on a set of criteria (usually supplied by either you or society) compared to others and
uses the results to determine your self-worth and social importance.

 Social invisibility: Here’s where we get into the marginalisation of people proper. Social
invisibility refers to individuals who have been put on the edge of society (WSC theme pun
apology imminent) and as a result are systematically overlooked by people within society (they’re
sort of considered as a true “outcast out-group”). It’s because of this that they are considered
invisible from society, their opinions and voices mattering little in actual affairs.

 amity-enmity complex | internalized oppression

 Amity-Enmity Complex: Unlike other contributors to theories on this list, Sir Arthur Keith was
Scottish anthropologist and anatomist (hardly anything to do with the sociological factors of his
work!) who argued that humans evolved as differing tribes, races and cultures. Those who were of
these same cultures were part of the in-group and were tolerated, whilst those who didn’t belong
were outcast in the out-group and could be subject to tensions or even hostility. In his 1948 work
A New Theory of Human Evolution, Keith was an avid proponent of this idea, building onto the
previous works on human tribalism (the development of our species in tribes and groups) from
authors such as Charles Darwin, Alfred Russel Wallace and Herbert Spencer.

 Internalized Oppression: This one is rather scary and it is somewhat sad to see the sorry state the
world is in because of this term’s existence. Internalized Oppression is rather straightforward and
is essentially when members of an oppressed group begin to use the methods of the oppressors on
themselves. The motivation is that these people who are mimicking the behavior of those in the
highly valued group (the oppressors) can move into their group and no longer be marginalised.
There are 3 main subcategories of this: internalized racism, internalized homophobia and
internalized sexism. All three can occur commonly in today’s society and it should be our priority
to cease this type of behavior, for it can even divide marginalised groups who are already receiving
enough pressure from external sources.

 black sheep effect | Robber’s Cave | Stanford Prison Experiment

 Black sheep effect (Caitlin): In the context of psychology the term black sheep effect refers to
the habit of group members have to judge likable/charismatic ingroup members in a more positive
light. Whereas, judging deviant ingroup members in a negative light. This essentially means that
you perceive likeable members of the in-group in a positive way, whilst those in the group who
don’t share a many desirable qualities or aren’t exactly fully aligned with the group are judged
more negatively than those outside the group who have the same qualities.

 Robber’s Cave (Caitlin): The 1954 Robbers Cave Experiment (RCE) is a very well known
example of Realistic Conflict Theory (RCT).

 Stanford Prison Experiment (Caitlin): The Stanford Prison Experiment was a social psychology
experiment conducted by Prof. Philip Zimbardo which took place in August of 1971, the aim of
the experiment was to investigate the psychological effects of power specifically focusing on the
power dynamic of prisoners and guards. Volunteers were recruited and told they would be
participating in a two-week simulation of prison, there were 24 volunteers selected to participate
in this experiment. The volunteers selected had no criminal backgrounds, psychological
impairments or medical problems. The volunteers also received $15 a day (the equivalent of $94
in 2018). The volunteers in this study were randomly assigned to the roles of “guards” or
“prisoners” in a mock jail, Zimbardo served as the superintendent of the jail. The experiment took
place in a 10.5-meter section of the basement of Jordan Hall. The “prison” had cell walls and an
entrance, there were only cots in each 1.8 x 2.7m cell for the “prisoners”. On the other hand, the
guards had a separate living environment from the prisoners with more luxuries and rest/relaxation
areas. The conclusions drawn from this experiment by Prof. Zimbardo are the following, “the
simulated-prison situation, rather than individual personality traits, caused the participants’
behavior. Using this situational attribution, the results are compatible with those of the Milgram
experiment (an experiment in which random participants complied with orders to administer
seemingly dangerous and lethal electric shocks to a shill) .” A positive “result” of this study was
the alteration of US prisons this being that when juveniles accused of federal crimes are no longer
housed with adult prisoners before their trials, due to the risk of violence against the juveniles.
There has been controversy over the ethics and scientific rigor of the Stanford Prison Experiment.
The treatment of “prisoners”, reliance on anecdotal evidence, coaching of the “guards”, using a
small unrepresentative sample and more were some of the issues with the experiment. A more
detailed informative video on the Stanford Prison Experiment can be found here
Here There be ____: Understanding the Margins of Society
 Mechanics of Marginalization

 Discrimination| colonialism | slavery | hegemony

 Discrimination (Caitlin): The prejudiced or unjust treatment of different groups of people or


objects. This can be seen on the grounds of sexual orientation, gender identity, race, age and more.

 Colonialism (Caitlin): Taking full or partial political control over another country, occupying it
with settlers, and exploiting it economically. Examples of this include the Hong-Kong by the
British, Macau by the Portuguese, Indonesia by the Dutch etc.

 Slavery (Caitlin): A system in which property laws are applied to human beings, this allows
individuals to purchase, sell and own other human beings as a de jure/ legal form of property.

 Hegemony: A term originating from Ancient greek city-states, Hegemony refers to the leadership
or dominance a social group or political party has over the rest of society. This is usually used in
diplomatic circles to denote the power struggles between the people and the politicians in countries
nowadays. The dominant group is known as the hegemon, while those within their sphere of
influence are referred to as the hegemonic.

 nepotism | endogamy | xenophobia | rankism

 Nepotism: If you’ve ever noticed those “Father and Son” or “Family Business” signs up around
your neighborhood, then you’ve seen an example of nepotism in action. Nepotism is the granting
of favors to relatives in various fields, though more commonly politics and businesses than normal
society. The term is actually based on the Latin term nepos, meaning “nephew”. History has it that
Catholic popes and bishops in the medieval and late renaissance ages would give their positions to
nephews, having taken a vow of chastity which eliminated the possibility of a direct lineage
takeover. Nepotism is where an employer might favour an application because that person is part
of their family or because they know someone from the family.
 Endogamy: Similar to polygamy and monogamy, endogamy deals with matters of love and
marriage. Specifically, in endogamic communities, marriage is only allowed by members within
the clan or group (as opposed to exogamy, where marriage with external clans is enforced). The
aim of endogamy is usually to preserve the bloodline of that clan and maintain the “purity” of the
group by not contaminating it with foreign or external influences. Scottish Highlanders, historical
aristocrats and the house of Hapsburg will easily speak of their pride in exogamic relationships.

 Xenophobia: Slightly different than racism (though the two have been easily confused in common
conversations), xenophobia is where one shows a prejudice or extreme dislike of people from a
certain nation, as opposed to people of a certain race. So despite the fact you may not be racist
against blacks, you might be xenophobic against Rwandans (whose majority-black population can
cause you being labelled racist when in reality you’re xenophobia). Xenophobia also refers to an
extreme distrust of that which is strange or foreign, as part of the ingroup perception towards the
outgroup.

 Rankism: You can probably guess the meaning of this one simply by the way the word is
constructed. Rankism is essentially racism but towards those of a different rank (or hierarchy) in
society. Rankism is essentially what gave rise to all the other “isms” when it comes to
discrimination in society. It’s the “somebodies” putting down the “nobodies” and it can occur in
all forms. The next section will explore more sub-categories of rankism.

 sexism | racism | ageism | ableism | elitism

 Sexism: This is one of if not the most pressing issue for many human rights activists around the
world. Sexism is the prejudice or discrimination of people based on their gender, which often gives
rise to stereotypes and gender roles. The roots of sexism are actually pretty old, for much of human
history society has been geared towards believing that males are “intrinsically superior” to females.
As a result, the roles of women in society were seen as subservient to men and their rights were
practically nonexistent until the 1800s. A common example of sexism in schools might be a teacher
or classmate assuming that girls are naturally emotionally unstable (not at all true in my opinion).

 Racism: Here’s the big one. Racism is the prejudice and discrimination directed against people of
a different race on the belief that your race is somehow superior. Race can refer to many things,
from skin colour to cultural background to even religion (though nowadays we tend to stick to the
former). Again, like sexism racism has its roots in history, where whites were seen as the
“dominating race” of humanity and as such “blacks, yellows, reds” and other races were considered
almost non-human or degenerate compared to them. In reality this is one of the greatest flaws of
our race. Why does a person's’ skin colour somehow dictate what privileges and rights they recieve
in society? In historical ideologies, it was believed that blacks were “intellectually inferior” (for
some inexplicable reason they thought that skin color equaled IQ).

 Ageism: Coined slightly later than all the other words (in 1969 to be precise) by Robert Neil
Butler, ageism is literally what you think it is: prejudice or discrimination based on their age. We
often do this type of stereotyping casually, without even realising that we’re being ageist. In the
employment sector, older people are overlooked because they’re perceived to be “unfit for work”
(despite the fact their years of experience would make them more valuable for the younger
recruits). There are often news stories about elderly people who challenge these stereotypes, being
more adventurous as they get older.

 Ableism: This is another major problem in the world today. Ableism is prejudice or discrimination
towards those who aren’t capable of performing the same tasks as “normal” people are due to some
sort of medical condition or birth defect (I’ll avoid using the word “disabled” since it can also be
perceived as a form of ableism). In society, these people are often shunned from certain positions,
being given certain rights or even from being treated like any other able-bodied person is.
Governments, NGOs and smaller communities alike are slowly beginning to solve ableism, since
some aspect of economic aid can be applied here. Nations with developed economies can often
afford the development of non-able friendly infrastructure (such as ramps for wheelchair users or
elevators for those on crutches).

 Elitism: This is something slightly more political but still inherently social. Elitism is the belief
or attitude that individuals who form an elite (a group with certain characteristics, income or
background) are more likely to contribute to society and as such should be given more power or
recognition by everyone else. In modern society, there are governments which employ this type of
thought concept, such as the totalitarian DPRK which believes that the Kim dynasty is the elite
ruling class.

 bullying | victimization | social dominance orientation

 Bullying: A common school presence, bullying is the act of trying to coerce, harm or intimidate
someone on some sort of illogical and irrational grounds (usually more emotionally based than
logical). Bullying often comes with the implied message of a serious power imbalance. The bully
dominates any interaction with the person being bullied, who has little or no power to stop this
interaction turning negative in nature (which it almost always does). About 1/3 of schoolchildren
have encountered bullying in their educational life and it can pose a serious problem (there have
been a slew of news reports about bullied child who took their own life). Bullying is one of those
activities where those who simply watch or ignore the events are sympathizing with the bully, as
opposed to standing up for the victim and halting the bully.

 Victimization: The result of bullying, racism and all other types of prejudice, victimisation is the
process of singling out a person or group of individuals for unjust or cruel treatment. It also applied
to victimology, the study of the psychological and sociological processes that occur when one
becomes the victim.

 Social dominance Orientation: Acronymized as SDO, Social Dominance Orientation is a


personality trait which forms a part of another theory, known as social dominance theory. The
theory simply states that humans have evolved to create group-based social hierarchies on a set of
criteria, which then served to influence evolutionary history (it actually draws some inspiration
from Marxist theories!). Social Dominance Orientation is essentially the psychological component
of that theory, developed by Jim Sidanius and Felicia Pratto in 1999. Social Dominance Theory is
the measure of social and political attitudes which will indicate how likely an individual is to
acclimatize to the theory of dominant hierarchical groups.

 Poverty and Social Stratification

 Gini index | Great Gatsby curve | Poverty Gap Index

 Gini Index (Caitlin): The Gini Index/ Gini Coefficient/ Gini Ratio is a statistical measure of
dispersion which represents the “spread” of wealth distribution in a nation. A Gini index value of
0 expresses perfect equality, where wealth distribution in a nation is completely equal. A Gini
Index value of 1 expresses maximum inequality for wealth distribution in a nation.

 Great Gatsby Curve: Alan Krueger, former chairman on the Council of Economic Advisers
introduced this measure to the world at a conference in 2012. On the horizontal axis of the graph
(an example of which is shown in the slideshow below), the Gini coefficient measure of a nation’s
wealth distribution. On the vertical axis is a separate yet related measure, known as the
“intergenerational elasticity of income”. Basically this refers to how much a percentage increase
in your family’s income will translate to your expected income when you grow up. The higher this
number is, the less social mobility you’ll have in the future; meaning that you’ll likely stay where
your family is right now in society. Rather frightening is the fact that some of the most developed
nations in the world, the United States and United Kingdom for example; have very low scores
when it comes to social mobility thanks to their high inequality. Unsurprisingly, we find that
Scandinavian countries such as Denmark or Norway are at the bottom of the curve, with their low
inequality meaning that social mobility is a perfectly possible occurrence.

 Poverty Gap Index: Introduced by the World Bank research group (that department of the world
bank meant to give statistics on practically every possible economic measure for every possible
nation in the world), the poverty gap index is a measure of the average shortfall of the population
from the poverty line (basically how far on average the poor people in a country are from no longer
being in poverty. A higher poverty gap index means that poverty is severe in the country and that
a large amount of money will be required to minimise the poverty gap index. If you multiply the
poverty gap index by the poverty line and total number of citizens, then that amount is roughly
how much money each person would need to earn annually to lift themselves out of the poverty
line. Here’s an example:

 A country has 25 million citizens and a poverty line of $750 a year, along with a poverty gap index
of 10%. The average increase of $75 per year per individual would eliminate poverty from this
nation, with the total amount of money required for the entire populace to no longer be in poverty
standing at $1,875,000,000 (good luck getting your country out of poverty, government officials).

 social class | social mobility | dissimilarity | division of labor


 Social class: Social class is a large part of social stratification (and I’m practically repeating myself
at this point in time, thanks WSC). It basically subjectively organises people in a society based on
their social and economic status. Historically, this may have taken the form of feudalism in
Medieval Europe (where kings and priests were at the top, followed by knights and other soldiers
and so forth). In Industrial Revolution Europe, this shifted towards a more economic lens, where
the upper classes paid the lower classes to do their work for them. The emergence of a middle class
during this time also expanded the social classes. In the modern society, our positions and income
still dictate which part of the social strata we fit into, though our rights and privileges aren’t exactly
hampered as much as they might’ve been in the past due to our position.

 Social mobility: Please take the time to watch this great Crash Course video about the concept, as
it explains it using visuals and diagrams much more comprehensive than I can.

 Dissimilarity: Differences between social classes essentially.

 Division of labor: Now here’s a term where I can call upon my communist comrades to support
the revolution and divide all labour equally amongst the people. Gone are the notions of capitalist
aristocrats or beggars, all will do their duty for the motherland in glorious Soviet Union!

 (throat clearing noise), sorry about that, I should probably get some medication for my communist
rants. Anyways the actual concept of division of labour was indeed a huge cornerstone for Marxist,
Capitalist and practically every other economic system. Division of labour refers to the factory-
level division of work into a number of tasks that are then completed by a separate group of
individuals before moving onto the next task. The concept has given rise to cheap labour costs and
lower retail prices as a result. The assembly line is a form of division of labour, eliminating the
need for skilled craftsmen who knew how to assemble every component of a product, instead
replacing them with low-skill workers who can repeat the same mind-numbing task day in day out.
Henry Ford is a common name used here, for it was he who first proposed the assembly line and
division of labour on his car production lines. Instead of giving 1 car to a skilled craftsmen to make
(which could take days or even weeks!), the conveyor belt was used to speed up work and divide
the tasks between “sub-groups” of workers (i.e engine technicians could work on installing the
engine, before chassis engineers worked on tweaking the body of the car).
 just-world fallacy | redlining | Davis-Moore hypothesis

 Just-world fallacy: If a criminal is convicted for a robbery that they committed poorly, most
people would say that they have deserved this punishment for their actions. That is the just-world
fallacy. For all you know, that robbery could’ve been against a corrupt business or a lying person
and that this was the only way that person could feed his family. The just-world fallacy is the belief
that people who are winning at the game of life have done something to deserve those rewards,
whilst does who are losing have done something to deserve those punishments. This bias is a lens
through which we see the world and we often have the same reactions to people who’ve found
themselves at the bottom of the chain or those at the top. There is no sort of invisible “moral
balance” or “karma” that berates or rewards those who have been morally good or bad, there is
merely the human mind and the deception it has wrought upon itself.

 Redlining: Used in the United States and Canada, redlining refers to the systematic prohibition
and denial of various services to residents of specific and often racially associated neighborhoods.
This can be done through the drastic raising of prices for those residents, or simply a law preventing
those inhabitants from partaking in the service.

 Davis-Moore hypothesis: In 1945, psychologists Davis and Wilbert E. Moore published a paper
that would serve as yet another addition to the works on social stratification. In the work, Davis
and Moore forwarded an attempted explanation of social stratification, giving the idea that this
was a functional necessity for the evolution of mankind. They also argued that the most difficult
jobs in society were the most necessary and that the motivation required to people to fulfill them
meant the highest compensation/rewards. Once these roles are fulfilled, the division of labour can
continue unabated and society could function properly.

 homelessness | slums | favelas | shanty towns | skid row

 Homelessness: As it implies, homelessness is the state of someone not having a permanent


dwelling, home or shelter to live in. The most likely reason is that these people were unable to find
a steady source of income to maintain or rent a home and as a result were forced to live in
temporary dwellings.
 Slums: Every city in the world has one of them (probably, I’m not sure what the Scandinavians
consider slums). A slum is a community of tightly packed and heavily populated buildings, all of
which are in a state of ruin or not properly maintained. This often means buildings made from
scrap materials (i.e leftover wood for the walls, grated metal for the roof and no solid foundations
to speak of). Slums are normally where the impoverished people end up living in the hundreds or
even thousands and they can be destroyed easily by fire, earthquake or even war.

 Favelas: A specific name for slums, favelas are Brazilian/Portuguese in origin and consist of a
large slum community that occupies the outskirts of towns such as Rio De Janeiro or São Paulo.
These favelas are once again the dwellings of impoverished people, packed tightly together and
constructed using makeshift (and at times stolen!) materials. The first favela appeared in what is
now Rio De Janeiro in the late 19th century by soldiers who had nowhere to go after the Canudos
War (a conflict between the state of Brazil and a group of some 30,000 settlers who’d made a
community called Canudos).

 Shanty towns: Similar to favelas, shanty towns are areas of tightly packed improvised settlements
on the outskirts of cities. They are often constructed using cardboard, plywood, corrugated steel,
sheet plastic and other salvaged materials.

 Skid row: Now we have a more specific example. Skid row is an area of Downtown Los Angeles
and it contains one of the largest populations of homeless people in the United States (anywhere
between 5,000 to 10,000 depending on reports). The area was defined in a court case called Jones
v. City of Los Angeles as the area east of Main Street, South of Third Street, west of Alameda
street and north of Seventh street. Skid Row’s history dates back to the remnants of the Industrial
Revolution, where impoverished loggers and gold miners who’d squandered their fortune came to
California in order to find a place to settle down, before realising that they could never afford
permanent housing.

 Race and Ethnicity

 segregation | self-segregation | ethnocentrism

 Segregation: One of the darkest periods of human history, segregation refers to the separation of
different racial groups in a community. When this word is spoken, you might have images of Cold
War America, where the anti-segregation movement had its roots. In America and South Africa,
this policy was notorious for its unequal treatment of citizens and the lack of privileges or rights
given to those being segregated. At the bottom of this page are a few images of segregation in
1960s America, notice the superior facilities that the whites had in comparison to the colored.

 Self-segregation: As the name implies, self-segregation is the separation of a group from society
with its own actions and for its own reasons. Instead of society shunning them, they’re shunning
society (sort of like an Uno reverse card or “no you” by the group in question). Because of this,
the group can then focus on offering their own services and preserving their traditions/customs.
For example, there remain many uncontacted tribes around the world who’ve undergone self-
segregation for fear that assimilation into society will erase their culture. In fact a more recent
example of such a segregation was Apartheid in South Africa, where the whites and blacks
practiced endogamy with their own races to preserve the tradition.

 Ethnocentrism: Consider the following analogy: ethnocentrism is to society what heliocentrism


is to astrology. Much the same way we perceive the sun to be the centre of the solar system and
base any following concepts off that fact, ethnocentrism is when we consider our own culture
inherently more important (sort of the center of all cultures) and therefore judge other cultures
based on preconceptions and standards in our own tradition. Coined by Polish sociologist (gee the
Poles sure do love attempting to explain us humans don’t they?) Ludwig Gumplowicz, the term
was later expanded upon by American social scientist William G. Sumner (and second on the list
of “most national contributions to wsc” is America). Ethnocentric people believe that they’ve been
superior to all races ever since their race came about and have been using the lens ever since. For
example, when China was under the rule of the Qing Dynasty, who were actually invaders from
Manchu (a mountainous northeastern province), ethnocentrism caused the public to actually look
down upon their “inferior” monarchy because of the belief that the true Chinese race was far
superior.

 tribalism | supremacism | reverse racism | eugenics


 Tribalism: The state or fact of being organised into tribes. Humans are naturally adept at creating
these tribes, as for much of our evolutionary history we’ve lived in small groups, nowhere near as
large as the societies that dominate our landscape today.

 Supremacism: As the name suggests, supremacism is an ideology which advocates that a


particular class of people are superior to all others and therefore should be given all the power to
dominate, control or even subjugate those who aren’t part of the supremacist race. This can range
from anything between religion, age, sex, cultural background, nationality or even appearance.
Perhaps two of the most notable examples of supremacism were “The White Man’s Burden”, the
belief in the 19th century that it was white people’s responsibility to teach the “inferior races” how
to be civil and more humane. The second example is in Nazi Germany, where Adolf Hitler
advocated the belief of the Aryan Herrenvolk (Aryan Master Race) in his dictatorial ruling over
the German people.

 Reverse Racism: As the name again implies (god these definitions really aren’t that subtle are
they?), reverse racism is the employment of color-conscious terminology and programs in
affirmative action to bring about the equality in society. In America and Europe, this movement
has gone under flak, with some saying that “Black Lives Matter” (being an example of reverse-
racism) is an attack on white people (clearly these people have never been introduced to jim crow,
slavery or really any of their history with the coloured people).

 Eugenics: Derived from the Greek word eugenes (meaning “well born”), eugenics is similar to
supremacism in that it is a set of beliefs or values that aim to improve the genetic quality of a
population by excluding the genetics of those groups perceived to be “inferior” or “undesirable”.
The term was first popularised by Sir Francis Galton, a British scholar and cousin of Charles
Darwin who in 1883 advocated that humanity enter selective breeding to ensure desirable traits
continued to exist. In the United States and later on in Europe, the idea continued to gain support
and by the 20th century, programs were in place to sterilise the “undesirables” to ensure their traits
would never be passed on (sterilisation is the act of depriving someone the ability to reproduce).
By the start of WW2, at least 65,000 people had already been sterilised under the eugenics beliefs.
Yet by the time war was upon them, the interest in sterilisation was lost. Nazi Germany (and later
Hitler) would endorse the use of eugenic principles to justify their atrocities, causing the concept
to lose all credibility and any governments to terminate funding for projects related to it.

 institutionalized racism | “race traitor” | “passing” |

 Institutionalized Racism: A form of racism practised in political or social institutions, from


government buildings to schools to even hospitals.

 “Race traitor”: It doesn’t take a rocket scientist (or social psychologist) to solve this one. A race
traitor is a reference to someone who is advocating beliefs, positions or attitudes thought to be
against the wellbeing or standards of their own race. For example, Germans who sheltered Jews
and prevented their arrests could’ve been labelled race traitors for their betrayal to the Aryan
master race (and they were brutally punished for it).

 “Passing”: Just the same way you might be able to “pass” a test by making the teacher believe
you can achieve a grade different to what your intellect on the subject might actually be, “passing”
in society means that you can be categorised as part of an identity group (meaning race, culture,
class etc.) different to their own. In many cases, this can often mean a change in social class or
caste based on new income statistics.

 Reparations

 Reparations: Paying back something for some sort of unspeakable debt. Slavery, war crimes,
murders etc all fall into reparation-worthy territory.

 Gender and Sexism

 institutional sexism | objectification | masculinity vs femininity

 Institutional sexism: Institutional racism but for sexism.

 Objectification: One of the most annoying problems our society on the margins faces,
objectification is the process of degrading someone to the status of a mere object in their existence.
More common amongst girls than boys, we often objectify people without realising the
implications and (admittedly I too) have been warned off by the person being objectified.
Objectification removes all sort of human characteristics from the person, such as their emotional
status, personality or capabilities. For more details on the 7 sub-categories of objectification, please
refer to this Wikipedia link for a start.

 Masculinity vs. Femininity: If you haven’t heard of these 2 words already, I would begin to
question where you have been for the past 10 years of your life. Masculinity and femininity are
taboo words in today’s society (or at least, social activist groups would like us to eliminate the use
of these words). Masculinity refers to qualities or attributes associated with men (i.e the inability
to display emotions such as sadness, being physically strong and courageous in the face of danger),
whilst femininity refers to those traits attached to women (emotionally unstable, physically weak
and uncertain of themselves). As is rather obvious, the feminine definition is clearly more
imbalanced than the masculine one. These two words can be used to shape someone’s character
and behavior, restricting their ability to display their true self in public, for fear of being called out
for their lack of masculinity or femininity.

 mansplaining | pay gap | gendercide | heteronormativity | glass ceiling

 Mansplaining: Mansplaining is essentially behaviour whereby a man explain a concept, idea or


other thing to a girl in a manner regarded as condescending or patronising. Apparently society
thought there was a need for this specific term, since it's a reflection of our cultural and gender
norms on males knowing more than females (not at all true by the way).

 Pay Gap: One of the greatest debates of our society is the pay gap. Essentially this is how much
women are being paid less than men (or in some cases, how other social groups receive less income
than others doing the same job). I suggest you go search the web for some specific statistics on
this.

 Gendercide: The systematic killing of members of a specific gender. For example, in India
gendercide takes the form of rape or assault against women (a very serious problem for the
country).

 Heteronormativity: Somehow we humans believe that the only "true and fitting" type of sexuality
is heterosexuality (relationships between people of different genders), hence why
heteronormativity is the societal term for people who believe in this concept. It often stems as the
root of homophobia and other prejudices against those of non heterosexual orientation.
 Glass Ceiling: An unofficially acknowledge barrier of sorts to prevent advancement in a
profession ,especially affecting those being discriminated based on allegations of race or gender.
Glass ceilings may take the form of certain requirements out of reach by members of minorities,
drastic facility quality reduction for women or even outright company bans of members moving
up in the ranks due to their race.

 gender identities | LGBTQ+ | toxic masculinity | intersexuality

 Gender identities: I will be honest here scholar, I do not enjoy having to explain this concept and
as such will leave you with this link as a starting point for further investigation.

 LGBTQ+: An initialism that is continually expanding to include ever more people in its
classification, LGTBQ+ stands for Lesbian, Gay, Transgender, Bisexual and Queer. There are
obviously many other letters, with the full initialism being LGBTTTQQIAA. Please investigate
that initialism with this link.

 Toxic Masculinity: Somehow we appear to believe that males are restricted in their range of
emotional expressions (the word “sad” or “crying” is unknown to their vocabulary apparently, very
false indeed). They are apparently supposed to be the “dominant alpha” in any relationship they
are in (again, very false and emotionally annoying at times) as well as the fact that anger is their
main mode of communication (a fist tends to do less harm than the words I find). Toxic Masculinity
should probably be avoided, since the males are somewhat at fault and also being victimized for
their inability to stick to the stereotype applied to them.

 Additional Terms to Explore

 second-class citizen | dehumanization | vagrant | alien | untouchable

 Second-class citizen: Consider this a formal attempt at discrimination. A second-class citizen is


someone who is systematically discriminated or prejudiced against within a state or other political
territory, on the grounds of their culture (race, background, religion, color etc.). Second class
citizens are limited in their legal rights, privileges, welfare access and socioeconomic opportunities
despite being actual citizens of a nation. In the United States, this was inherently a problem with
segregation, whilst the EU currently has problems with their migrant/refugee second-class citizens.
 Dehumanization: What do slavery, objectification and dehumanisation have in common?
Answer: they deprive humans of their human qualities. Dehumanisation refers to the process of
degrading and treating individuals as though they are no longer human, stripping them of their
individuality and ability to make their own decisions. The basic definition relies on treating them
as though their mental capacity does not allow them to be human (which applied to slaves, who
weren’t allowed to learn the skills and information to gain “human” brains).

 Vagrant: A person without a settled home who wanders around from place to place, begging for
money to the general public who pass by him/her. Vagrants was (and still is) a derogatory term
used by society to denote people of “unclean” origin in addition to beggars who appear to not
understand human etiquette.

 Alien: An extraterrestrial being of unknown origin. Just kidding, in this context an “alien” refers
to an entity or individual which is foreign to a country and as such is referred to by the term when
in customs or immigration.

 Untouchable: In the Indian caste system, untouchables were the lowest band of people. They were
unable to own property, drink from public wells, partake in public events and at times were killed
simply for stepping on someone’s shadow (apparently it was believed that this desanctified the
person).

 homo sacer | lumpenproletariat | baekjeong | burakumin

 Homo sacer: A latin term meaning “accursed man” or “the sacred man”, Homo Sacer was a
Roman term used in law to refer to individuals who had been banned from public life and were
liable to be killed by anyone (free of trial). They were not however, allowed to be used for religious
sacrifices. The only way to become a homo sacer was to break an oath, which at the time meant
invoking the wrath and punishment of several deities if the oath was broken (thus making the
oathbreaker the property of the gods whose oath they had broken).

 Lumpenproletariat: Ah finally, a true Marxist term whose meaning I will happily go on about.
Simply put, the lumpenproletariat is the untouchable caste for the Marxist theory. This term was
used to refer to individuals in the “unthinking” lower strata society by Karl Marx and Friedrich
Engels in 1840. This sort of class was often utilised by revolutionary and counter-revolutionary
forces, despite the fact that their only worth when establishing or toppling a government was to
gain more support and bodies to throw at enemy forces. Their revolutionary potential was
dismissed and common people in this group included vagabonds as well as prostitutes.

 Baekjeong: the untouchables of South Korea. Kindly investigate their specifics on your own (I’m
really tiring at this point in time).

 Burakumin: Literally translating to “village people” in Japanese, Burakumin were at the bottom
of the social strata in Japan and as such were heavily discriminated and ostracised throughout
history. In the Japanese feudal era, these were people who were part of outcast communities,
composed of those who had been tainted by death (executioners, butchers, tanners, slaughterhouse
workers to name a few professions). They lived in their own villages, hamlets or ghettos and were
often separated from the rest of society who feared mingling with them.

 cagot | ragyabpa | bui doi | tanka | osu | akhdam | bitlaha

 Cagot: Now we’re getting into the actual minority groups and their histories (something I would
love to explain in more detail had I not been writing this late at night). We start our minority
journey in western France and northern Spain with the Cagots. A group that existed about as far
back as the 11th century, cagots are neither ethnic nor religious in nature; rather they were
identified as such thanks to their ancestry. The actual reasons for the persecution and suffering of
this group are unclear, some believe that they were lepers, cretins, heretics and even cannibals.
The list of their treatment is rather astonishing and is extremely detailed. They had to live in
communions separate from other people (called cagoteries), they had to announce their presence
by ringing bells, they could only enter churches from certain entrances, they received communion
(the Lord’s Supper) on the end of long wooden spoons, they couldn’t partake in any social or
political activities, hold cabarets, my god they couldn’t even touch the food in markets or sell their
own.

 Ragya Bpa: A social class in Tibet (or the Tibetan Plateau, as the Chinese government prefers to
call it). These were the untouchables of their society (dear me we humans loved giving this title
out in any form didn’t we?). They weren’t exactly so bad, they were similar to Burakumin in that
they performed “unclean” work. Interestingly however, these people were also given subdivisions
for their class. A goldsmith was at the highest sub-division for the ragya bpa, whilst fishermen
could find themselves in the middle with executioners at the bottom. Escape from this class was
impossible and it was considered unclean to even interact with them.

 Bui doi (Caitlin): Bụi đời is a Vietnamese term which means “the dust of life”. This term
originally referred to the starving people from the countryside who sought refuge in towns during
the 1930s. The term bụi đời currently refers to people, mostly men who live on the streets or who
are wanderers. A similar term trẻ bụi đời or “young vagrants” currently refers to street children or
adolescent gang members, the term is intended to strike an image of a purposeless child who moves
about like dust. Bụi đời has a more racial connotation in the west, this is because of the connection
to the musical Miss Saigon (which I highly recommend watching- Caitlin). In Miss Saigon, there
is a song of the same name as this term. In the song, this term is used to describe the “offspring”
of American G.I’s and Vietnamese Mothers who were abandoned at the end of the Vietnam War.

 Tanka: Alright then, let's clear up some confusion. Tanka in this context does not refer to the
Japanese poem with a specific amount of syllables per line (though I'd like to read a tanka about
the tanka!). The Tanka are instead a minority group of people who live in Southern Chinese
provinces such as Guangdong, Guangxi, Fujian, Hainan, Zhejiang as well as the SARs of Hong
Kong and Macau. Traditionally, these people lived on junks in the coastal areas of those provinces,
though nowadays many have moved to inland dwellings yet still retain some of their cultural
practices, which cannot be found in any other Chinese ethnicity. Historically, the Tanka was a
marginalised minority known as "sea gypsies" by the Chinese and British, who were able to trace
their lineage to the ancient Baiyue people (indigenous non-Chinese sea nomads who lived on the
coast). Interestingly however, the Tanka were never really prosecuted or discriminated against
much the same way African-Americans might have been in the US. Indeed for some Chinese
dynasties (mainly the Ming and Qing), they were exploited for their fishing businesses and weren't
exactly included in Han culture. Yet when the Portuguese and British first landed in China, it was
the Tanka whom they found the most likeable due to the fact that the Tanka people were willing
to interact with them. In fact the British even set up special schools for the Tanka in order to ensure
their educational well-being (at least in Hong Kong they did). It should be noted that a small
number of these "boat dwellers" can also be found in Vietnam, where they are called the Dan
people under the Ngai ethnicity.

 Osu: If you first thought about the musical game when this term popped up, I do not condone you
for that consideration (indeed for some time I had no idea what this term meant in relation to
society). Now however, a quick Google refining search and we get to the Osu Caste System. Native
to Igboland, a region in Southeastern Nigeria where the Igbo ethnic group live, the Osu Caste
System is an ancient practise of that region wherein people are discouraged from interaction and
marriage with the Osu. Osu in the Igbo language translates roughly to "outcast" and these people
are fully dedicated to the deities (or Alusi) of Igboland. As such, the osu are considered inferior
beings and are separated from the Nwadiala or diala (Igbo for "real born"). These people are made
to live in shrines or marketplaces and have no interactions with the real born, so it's little wonder
that many people in this social strata have fled to other countries seeking refuge from their
discrimination and harsh treatment.

 Akhdam: Finally a proper term (I’ve seriously got no idea about the relationship of the previous
two). Translating to “the servants” in Arabic, Al-Akhdam is a term used in Yemen to refer to a
minority social group that is at the bottom of the caste ladder and as such are socially segregated
from the rest of society. We don’t actually know for sure where the Al-Akhdam came from, but
the main theory is that they were companions of Abyssinian conquerors who occupied Yemen in
the pre-Islamic period who were actually descendants of Nilotic Sudanese. In the modern age,
these people number between 500,000-3,500,000 strong and take up menial jobs in society (shoe-
cleaning, sweeping, cleaning latrines as well as shoe-making to name a few). They reside in slum
districts which are isolated from the rest of Yemeni society and we don’t really know why these
people have been discriminated so heavily, after all they are still Arabic-speaking Muslims who
live in Yemen.

 Bitlaha: The final term for Social Studies (thank goodness its finally over and the additional cases
are fairly less sensitive), bitlaha is a South Asian term used to refer to social punishment for anyone
caught breaking the rules of exogamy and endogamy. The concept has been used by the Santals of
India and the Satars of Nepal, who call themselves hod (translating to “human beings”). They
consider Bitlaha as an outcast, disorder, polluted or unclean person. Once a person has attained
this status, they are no longer considered part of their community and as such are shunned (and in
some cases exiled) from their home village. They can however, remove this status by having the
pancha (a male politician in Hod society) impose upon them a severe penalty.

Additional Cases & Guiding Questions


Merging with the Masses
 Consider the so-called Asch Conformity Experiments and the conclusions drawn from them
about how and why people might conform to the opinions of those around them. Are there
ways in which you might critique the original experiment or the resulting paradigm? Discuss
with your team: when, if ever, is it good to conform?

 I’m going to skip the lengthy explanation of the Asch Experiments themselves and instead redirect
you to the specific prompt about them somewhere on this page. There are several critiques I would
make about the original experiment and the resulting paradigm (though many of them have already
been written about by previous critics of Solomon Asch). The first is the heavy reliance on stooges,
people who were in on the “correct answer” that would pressure the naive participant to conform.
In reality, we don’t have a large number of people who know what they’re going to conform to as
one collective unit, rather the Asch Experiment should have experimented on only 2 or 3 stooges
at most (not 7!). Another critique is the necessity of the participants to declare their answers out
loud, something very rare in society (how often do you find a person who shouts out what store
and product they’re going to?). In short there are many criticisms of the Asch Experiment and it is
no way a definite paradigm that we should apply to today’s society (remember, back then the
concepts of the internet, mobile phone and applications didn’t exist!).

 Conformation is one of those activities which we humans undertake naturally (you might’ve
conformed to your peers by following their example of reading this website, for which I thank
you!). There aren’t many cases of when conforming might be the “right thing” to do that I can
think of. Since conforming relies on the assumption that the people you’re conforming to are
correct or that this will prevent any negative consequences; there is no 1 type of situation in which
conforming is guaranteed to prevent harm.

 Look into the work of Project Implicit. Should its findings be applied in everyday life, and,
if so, how? How might someone critique their work? If you have time, try taking one of the
tests on this page, and, if you feel comfortable sharing the results, discuss the experience with
your team afterward.

 So far, we’ve explored how conscious processes can lead to social strata placement and
marginalisation. Project Implicit however, deals with what lies beneath the surface of our thoughts.
As the name implies, the initiative by several researchers from different US universities aims to
collect data about the hidden cognitive biases and processes which impact our attitudes toward
certain groups. The findings of this project are indeed capable of being applied in everyday life. If
societies were to understand why they seem to “naturally” bias against or towards a certain group,
then their levels of empathy, sympathy and even tolerance to that group might prevent further
oppression. In order to critique their work, one might bring up the fact that the range of questions
is very limited in terms of racial scope (the basic races and groups are included, but not enough
are there to fully scope out a person’s biases).

 I have yet to take the tests on this page (perhaps some time I will), so have yet to feel the experience
of exploring my social unconsciousness (if that’s even a phrase).

 Fashion (voluntary and involuntary) is strongly linked to group formation and membership.
Does the rise of the often unisex “normcore” movement in recent years point toward a more
inclusive fashion industry, or is it just another way for the “ingroup” to spend money to
differentiate itself? Discuss with your team: does fashion perpetuate perceived differences
between genders, races, and cultures?

 Please take my advice scholar and investigate this concept for yourself, for my knowledge on
fashion alone is deplorable (I struggle to name the types of clothes we wear beyond “shirt, pants,
shoes, hat, socks).

 Have you ever known someone who didn’t want to dance—whom someone else tried to force
onto the dance floor? Consider this article by the scientist Henry Reich, then discuss with
your team: when is it right to encourage someone to do something they don’t want to do?
Would you ever pressure someone to go to a party? Should we always let people opt out of
social activities, or are there times when it is appropriate to intervene “for their own good”?
You may also want to look at expectations around cosmetics, marriage, and vacation days.

 Practically every WSC Ball, I’ve seen someone who refused to strut their stuff on the dance floor,
before being promptly dragged onto said dance floor by their peers and pressured into showing off
their moves (all of which range from doing the Daniel to world-class performance level). The
article written by Henry Reich is a rather provocative and eye-opening one and I wholly suggest
that you read it thoroughly before continuing on this page. There are some times when we need to
encourage people to do something they don't want to do. If their current actions will cause harm
to some, then perhaps they need to be made aware of that so they realise the consequences of their
choices. I would never pressure someone to go to party, seeing as I myself would probably not
attend that party in the first place. That person’s schedule and choice when it comes to their
recreation are not mine to control (fun fact: I see more benefit and fun in studying for WSC than
going to a party). People should be allowed to opt out of social activities, we should not impose a
restriction on how long they need to stay before they can leave, there are times when your brain
simply says that you’ve had enough socialisation and some alone time would be highly valued.
There probably aren’t times when intervening at social functions is “for the greater good”.
Beyond the Norm
 Consider schools dedicated to the education of highly gifted children—such as the Mirman
School in California, which admits only children with IQs of at least 145, and its many highly
selective counterparts around the world, from Kazakhstan to Israel. Then, discuss with your
team: should high-achieving learners be separated, in whole or part, from other students? Is
there a difference between exclusion and exclusivity, and is one more acceptable than the
other? Are people too quick to judge programs of this kind - and, if so, what might be
motivating their judgments?

 What an interesting notion: separate the more capable among us to learn more so that they can lead
the rest of us when they grow up. A ridiculous notion whose creator I would very much like to sit
down and talk with (though perhaps the Mirman school director will suffice). High-achieving
learners shouldn’t be separated from their peers, instead they should be given more extensions in
addition to the encouragement to teach their own peers (we get to the finish line together, or we
don’t get there at all). Sure they can have their own unique classes within their schedule that allow
them to exploit their increased intellect, but they should never be detached fully from their school
community just to enhance their own experience. Exclusion refers to the act of excluding people
from a certain community, whilst exclusivity refers to the state of something being reserved for a
certain group of people (basically the two are opposites, one shuns groups whilst another only
accepts them). Exclusivity is generally more acceptable than exclusion since the latter generally
involves more aggressive actions to achieve itself.

 Consider the phenomenon of self-segregation on school campuses. Do you see it at your own
school? Is it something administrators should take measures against, and, if so, what kinds
of measures? Does it matter what the reason for the self-segregation is—for instance, among
gender, ethnicity, religion, age, or other values?

 Self-segregation is one of those events which our society needs to prevent from happening. We
need to avoid people of already marginalised groups from segregating themselves in environments
that are meant to be inclusive and accepting of all. As my school is an international school, this
doesn’t happen at all and we encourage people of minorities or unique races, religions and
ethnicities to express themselves instead of shunning their unique heritage. School administrators
should be taking action on this, probably by preventing it when it occurs and promoting groups
(such as service ones) which bring to light the experiences of these people.

 For every superstar in the NBA, a player is warming the bench. Do such players deserve
more credit than they receive—and is it ever right for them to ask for more recognition or
for more playing time? Are there similar hierarchies in other sports, or in the professional
world, and are they ever unfair? Discuss with your team: should special talents lead to special
treatment?

 Such players are probably getting the credit they deserve, it does take a certain amount of skill and
practice (as the website suggests) to be a bench player in a professional game of any sport. It might
be right for them to ask for more recognition or play time, especially if the coaches have ignored
them for several games in a row (or if they made an amazing comeback while no longer being on
the bench). Such hierarchies exist in many other sports as well, you probably don’t remember the
caddies or drivers who give those golfers the clubs and locations they need, nor does society
celebrate the referees or other supervising officials who make sure the game is fair and honest.
They aren’t unfair so to speak, indeed society has far more unfair hierarchies.

 Special talents might lead to special treatment, but not exclusive attention. Just because someone
is extremely skilled at handling a football does not mean they need to enter a specific class of
highly-skilled athletes. Likewise, people with an inability to handle a ball shouldn’t need to be
placed in a lower class to learn the basic throwing and catching skills (it’s their choice whether or
not this treatment is right for them).

 Explore social programs in countries with very low rates of homelessness and destitution,
such as Finland, Denmark, and Japan. What is particularly effective about their
approaches? How would you advise countries—or cities, such as San Francisco—that are
experiencing very high rates of homelessness?

 Social Programs in Finland, Denmark and Japan mainly centre around preventing homelessness
by ensuring those who are at risk of losing their homes earn enough money to keep them. The
government does not have to rely on setting up “homeless shelters” so much as the US has had to,
instead they can afford to pay adequate pensions or provide housing programs for the elderly,
jobless or invalids to ensure they can live under a safe roof. I’d probably advise countries such as
San Francisco to apply the same philosophical approach: stop the problem at its root. It doesn’t
matter that the city mayor has proposed opening hundreds more shelter beds, the number of
homeless people will just keep rising as long as they aren’t given decent retirement pensions, social
welfare privileges or immediate job placement priority.
Toward an Integrated World
 Some researchers have concluded that companies (and even countries) are more likely to
select women for positions of leadership when they are already in decline—making it more
likely that these women will fall off what they term the “glass cliff” of failure. Discuss with
your team: to avoid such a phenomenon, should companies and countries be required to
alternate regularly between male and female leaders?

 Let’s get into this somewhat precariously dangerous explanation. The “glass cliff” is an economic
term and social phenomenon that occurs when companies which are already in decline (either
facing mass job layoffs, store closures or stock plummets) assign woman to positions of leadership
in their workplace. As a result of their downward trend, the woman have more pressure to
somehow salvage the company and turn it around, causing many to crack and the company to fail
even more. Thus while the “glass ceiling” prevents woman from moving any higher in the
hierarchy of job positions, the “glass cliff” puts woman in the greatest positions of power, yet the
precarious situation means they’re more likely to fall back down to where they started. The term
was first coined by Michelle Ryan and Alexander Haslam in 2003, researches at the University of
Exeter who were investigating whether women leaders had a negative impact on company
performance. What’s even more amazing is that the women who do fall victim to the “glass ciff”
are more likely to take huge risks in the world of business to make sure they (or their company)
doesn’t fall from its current situation.

 To prevent such a phenomenon from occurring, companies should pool together the collective
know-how of woman and men on the team of executives, as opposed to simply resorting to the
failsafe of giving women the top decision making spots.

 Is Canada really inclusive, or does it just have excellent branding? Discuss with your team:
what is the best way to measure the inclusiveness of a country or society? What countries
would you consider to be the most inclusive, and what do you think has made them that way?

 Ah Canada, that land which is stereotyped for not having any stereotypes in its everyday lingo and
yet is still regarded as one of it not the most inclusive place in the world. Canada’s inclusiveness
has always been a trademark of the country and this is technically still true, thought there are some
who continue to live on the margins in the society, they aren’t as nearly oppressed or ignored as
those in say the United States or continental Europe. Yet the country still faces challenges, wealth
inequality amongst the people still creates a “rich and the rest” environment. Youth unemployment
figures still stand at a somewhat concerning amount. I’d say that Canada isn’t entirely inclusive,
but simply does a better job of normalising the gaps in society to the rest of the world while at the
same time making efforts to eradicate those gaps. There isn’t exactly one way to measure the
inclusiveness of a country or society, merely looking at a conglomerate of statistics will give you
a rough idea of how well a country is doing in terms of allowing all groups in its society to
participate in economic, political and social activities.

 In a world in which many countries (including the United States) are highly religious,
atheists—non believers—arguably encounter challenges fitting in with mainstream culture.
Consider this article about the rise of an atheist rights movement, then follow up on its
claims. Is such a movement justified?

 It seems somewhat shocking that members of our society have been marginalised, harmed and
even killed over the simple fact that they choose to follow no religion and believe in no god. The
fact that something as seemingly normal and acceptable as atheism is now becoming a rallying
banner for another civil rights movement, similar to LGBTQ communities or feminist activists.
Such a movement might be justified with the current state of the world, with so many countries
being majority Christian, Hindu, Muslim or Jew populations (surprisingly, the most populated
country in the world has the highest percentage of people who are atheists).

 Ancient Greeks mocked people with physical differences; today, we have disability rights
movements and disability studies. How have people with disabilities such as deafness,
blindness, and lack of mobility advocated for inclusion in mainstream society? Discuss with
your team: do you support those who may wish to resist that inclusion?

 Simply by continuing to exist and by refusing to be put down, these people who were born (or
were medically affected) with special conditions have managed to advocate for their inclusion in
mainstream society. Once scientists and doctors found the “cures” to their conditions, they could
now become normal members of society with the same social mobility and opportunities that
others had around them. Of course there are those who think that their condition makes their life
more unique and in some cases more enjoyable and have resisted being assimilated into normal
society through the use of implants and other medical technologies. Their wishes are to be
respected, for it is their own lives whom we might be corrupting or ruining unintentionally in some
way.

 Are women held to different standards than men in the pursuit of elected office? Discuss
with your team: can elections ever be counted on to produce fair results, if people have
underlying prejudices toward one or more groups?

 Women are indeed held to different standard than men in the pursuit of elected office and other
political positions. Not just their clothes (as the Vox article linked suggests), but also by their own
abilities in congressional debate, ability to keep their emotions in check or even their ability to fall
in line with what their populist nation has to say. Elections have always been touted as the way for
a government to produce fair results, so that the people can be lead by a government that was
nominated for their own benefit. Now however, with the inclusion of more groups into society, the
political sphere is still resisting the assimilation of more groups (minorities, genders, races and
even ages). Elections for now will have to do, but candidates with prejudices against them will
likely find their journey to the ruling party laden with more obstacles.

 In India, a woman just scaled a mountain that until now had been reserved for men only.
Discuss with your team: is it ever right to restrict access to a place to members of one gender?
How about to one age group, or to one religion or culture? If your answers are different for
different categories, what makes one restriction okay and another not?

 Is is somewhat unsettling to find that our people have found some sort of theological reason to ban
others from certain natural or man-made sites. Indeed certain library sections in the US were off-
limits to blacks during segregation, whilst temples in Ancient civilisations were off-limits to
women. It never seems right to restrict access to a place to members of any group, gender, religion
or culture. We humans need to share this planet (as hard as that seems and as harder as its getting!)
and by being born here we have the right to any accessible location. Our maps and travels should
not need to be drawn by our race, religion, culture or gender.

 Sometimes, people are excluded from society for (ostensibly) the common good; for example,
criminals are placed in prisons. In some of Norway’s prisons, however, the criminal justice
system aims to keep prisoners as part of society. Discuss with your team: to what degree
should we prioritize reintegrating prisoners with the rest of the world?

 I seriously suggest looking at the article. The photos attached are stark in contrast and extremely
eye-opening when it comes to how different societies treat their criminals. Norway, Sweden and
Denmark have been praised for their rehabilitation concepts and the resulting humane design of
prison (if I was ever convicted for a crime, I’d enjoy my time in a Norwegian prison). We should
highly prioritize reintegrating prisoners with the rest of the world. These people were probably
misguided in their actions or simply the victim of an unfortunate series of events. They are still
humans, still members of society and still capable of making up for their crimes. Prisons should
not treat them as if though they are an abomination from the forces of evil itself, instead they
should attempt to rehabilitate and serve as a place where these people can continue to live and
learn the error of their ways. Perhaps with enough time and the correct facilities (such as those in
Norway), these convicted persons can see the light of day and the norms of society once again.

You might also like